You are on page 1of 84

eMRCS 19/07/2015

0/3
Question 1-3 of 41

Theme: Management of biliary disease

A. Acute laparoscopic cholecystectomy


B. Delayed laparoscopic cholecystectomy
C. Percutaneous cholecystostomy
D. Elective cholecystectomy and intra operative cholangiogram
E. Endoscopic retrograde cholangiopancreatography
F. Choledochoduodenostomy
G. Bile duct excision and hepatico-jejunostomy
H. Operative cholecystostomy

For each scenario please select the most appropriate management option. Each option may be used once, more than once or not at all.

1. A 72 year old lady underwent an open cholecystectomy 12 years previously. She has been admitted since with 2 episodes of cholangitis
and stones were retrieved at ERCP. She has just recovered from a further episode of sepsis and MRCP has shown further biliary stones.

You answered Percutaneous cholecystostomy

The correct answer is Choledochoduodenostomy

A patient with long standing common bile duct stones is at risk of developing duct fibrosis and ductal disproportion. This can result in
impaired biliary drainage. Not only may further stones be formed in the bile that is present, but because of the ductal disproportion the
tendency will be for the stones to accumulate (rather than pass spontaneously, as would usually be the case post ERCP and
sphincterotomy). A biliary bypass procedure is the standard method dealing with this and a choledochoduodenstomy is one procedure that
can be used.

2. A 26 year old women is admitted with acute cholecystitis of 24 hours duration. LFT's are normal and Ultrasound shows a thick walled
gallbladder containing stones.

You answered Choledochoduodenostomy

The correct answer is Acute laparoscopic cholecystectomy

This is an ideal case for an acute cholecystectomy, provided that surgery can be undertaken promptly. After 48 -72 hours the patient
should receive parenteral antibiotics and delayed cholecystectomy performed.

3. A 32 year old lady is seen in the outpatients. She has had multiple episodes of biliary colic and ultrasound shows thin walled gallbladder with
multiple calculi. Her ALT is slightly raised but other parameters are normal.

You answered Endoscopic retrograde cholangiopancreatography

The correct answer is Elective cholecystectomy and intra operative cholangiogram

The easiest option is to perform an intraoperative cholangiogram. It is unlikely to reveal any stones. If is does then either laparoscopic bile
duct exploration or urgent ERCP can be performed. An MRCP pre op is an alternative strategy.

Next question

Biliary disease

Diagnosis Typical features Pathogenesis

http://www.emrcs.com/question/question.php?q=0 1/3
eMRCS 19/07/2015

Gallstones Typically history of biliary colic Usually small calibre gallstones which can pass through the cystic duct. In Mirizzi
or episodes of chlolecystitis. syndrome the stone may compress the bile duct directly- one of the rare times that
Obstructive type history and cholecystitis may present with jaundice
test results.

Cholangitis Usually obstructive and will Ascending infection of the bile ducts usually by E. coli and by definition occurring in a
have Charcot's triad of pool of stagnant bile.
symptoms (pain, fever,
jaundice)

Pancreatic cancer Typically painless jaundice with Direct occlusion of distal bile duct or pancreatic duct by tumour. Sometimes nodal
palpable gallbladder disease at the portal hepatis may be the culprit in which case the bile duct may be of
(Courvoisier's Law) normal calibre.

TPN (total Usually follows long term use Often due to hepatic dysfunction and fatty liver which may occur with long term TPN
parenteral nutrition) and is usually painless with non usage.
associated jaundice obstructive features

Bile duct injury Depending upon the type of Often due to a difficult laparoscopic​ cholecystectomy when anatomy in Calots triangle is
injury may be of sudden or not appreciated. In the worst scenario the bile duct is excised and jaundice develops
gradual onset and is usually of rapidly post operatively. More insidious is that of bile duct stenosis which may be caused
obstructive type by clips or diathermy injury.

Cholangiocarcinoma Gradual onset obstructive Direct occlusion by disease and also extrinsic compression by nodal disease at the porta
pattern hepatis.

Septic surgical Usually hepatic features Combination of impaired biliary excretion and drugs such as ciprofloxacin which may
patient cause cholestasis.

Metastatic disease Mixed hepatic and post Combination of liver synthetic failure (late) and extrinsic compression by nodal disease
hepatic and anatomical compression of intra hepatic structures (earlier)

A gallbladder may develop a thickened wall in chronic cholecystitis, microscopically Roikitansky-Aschoff Sinuses may be seen

Image sourced from Wikipedia

Save my notes

Please rate this question: Next question

http://www.emrcs.com/question/question.php?q=0 2/3
eMRCS 19/07/2015

Send feedback to the author

Question stats

Average score for registered users:

1 35.5%
2 61.9%
3 48.3%

Search eMRCS

Search term Go

External links

Suggest a link

Dashboard

Overall score: 0%
1-3 0 / 3

All contents of this site are © 2015 E-Medical Revision Ltd Back to top

http://www.emrcs.com/question/question.php?q=0 3/3
eMRCS 19/07/2015

3/3
Question 4-6 of 41

Theme: Management of pancreatic malignancy

A. Gastrojejunostomy
B. Pancreatoduodenectomy
C. MRI guided pancreatic stent
D. Endoscopic pancreatic stent
E. Duodenoduodenostomy
F. Pancreatic radiotherapy

Please select the most appropriate treatment for these patients with pancreatic cancer. Each option may be used once, more than once or not at
all.

4. A 40 year old lady presents with new onset dyspepsia. She is diagnosed as having a localised cacinoma of the pancreatic head.

Pancreatoduodenectomy

Theme from January 2013 exam


Localised carcinoma of the pancreas is treated with a pancreatoduodenectomy, the eponymous name for this is a Whipples procedure.
Newer variants of the procedure include pylorus preservation. Adjuvent chemotherapy is often used.

5. A 67 year old lady presents with jaundice and abdominal pain. Her investigations show a dilated common bile duct, a carcinoma of the
pancreatic head compressing the pancreatic duct. Her liver contains bi-lobar metastasis.

Endoscopic pancreatic stent

Jaundice associated with pancreatic cancer is best managed with a stent. These are usually inserted at the time of ERCP. Consideration here
should be given to the use of a metallic stent (which is contra indicated where resection is contemplated).

6. A 67 year old lady presents with symptoms of persistent vomiting. Her investigations show gastric outlet obstruction from a carcinoma of
the pancreatic head. Her liver contains bi-lobar metastases.

Gastrojejunostomy

Gastric outlet obstruction from pancreatic cancer is best managed with a surgical bypass procedure or a duodenal stent (if the disease is not
resectable or curable).

Next question

Pancreatic cancer

Adenocarcinoma
Risk factors: Smoking, diabetes, Adenoma, Familial adenomatous polyposis
Mainly occur in the head of the pancreas (70%)
Spread locally and metastasizes to the liver
Carcinoma of the pancreas should be differentiated from other periampullary tumours with better prognosis

Clinical features
Weight loss
Painless jaundice
Epigastric discomfort (pain usually due to invasion of the coeliac plexus is a late feature)
Pancreatitis
Trousseau's sign: migratory superficial thrombophlebitis

http://www.emrcs.com/question/question.php?q=0 1/3
eMRCS 19/07/2015

Investigations
USS: May miss small lesions
CT Scanning (pancreatic protocol). If unresectable on CT then no further staging needed
PET/CT for those with operable disease on CT alone
ERCP/ MRI for bile duct assessment
Staging laparoscopy to exclude peritoneal disease

Management
Head of pancreas: Whipple's resection (SE dumping and ulcers). Newer techniques include pylorus preservation and SMA/ SMV resection
Carcinoma body and tail: poor prognosis, distal pancreatectomy, if operable
Usually adjuvent chemotherapy for resectable disease
ERCP and stent for jaundice and palliation
Surgical bypass may be needed for duodenal obstruction

Save my notes

Please rate this question: Next question

Send feedback to the author

Question stats

Average score for registered users:

4 83.8%
5 73.1%
6 63.7%

Search eMRCS

Search term Go

External links

http://www.emrcs.com/question/question.php?q=0 2/3
eMRCS 19/07/2015

Suggest a link

Dashboard

Overall score: 33.3%


1-3 0 / 3
4-6 3/3

All contents of this site are © 2015 E-Medical Revision Ltd Back to top

http://www.emrcs.com/question/question.php?q=0 3/3
eMRCS 19/07/2015

1/3
Question 7-9 of 41

Theme: Management of jaundice

A. ERCP
B. MRCP
C. Percutaneous transhepatic cholangiogram
D. Laparotomy
E. Laparotomy and formation of hepatico-jejunostomy
F. Laparoscopic biliary bypass
G. CT scan

For each of the following scenarios please select the most appropriate next stage of management. Each option may be used once, more than once
or not at all.

7. A 65 year old man is admitted with jaundice and investigations demonstrate a carcinoma of the pancreatic head. An ERCP is attempted but
the surgeon is unable to cannulate the ampulla.

You answered Laparotomy

The correct answer is Percutaneous transhepatic cholangiogram

Theme from September 2012 Exam


Cancer of the pancreatic head will cause obstructive jaundice and intrahepatic duct dilatation. When an ERCP has failed the most
appropriate option is to attempt a PTC. This procedure is always preceded by an ultrasound (which presumably this patient has already had
or they would not be undergoing an ERCP). Prior to performing the PTC it is important to stage the disease and establish resectability or
not. This is because the PTC drains frequently dislodge and fall out. It is usually desirable to pass a stent at the time of doing the PTC to
mitigate the effects of this problem.

8. A 48 year old lady is admitted with attacks of biliary colic and investigations show gallstones. A laparoscopic cholecystectomy is performed.
The operation is technically challenging due to a large stone impacted in Hartmans pouch. Following the operation she fails to settle and
becomes jaundiced and has bile draining into a drain placed at the surgical site.

ERCP

In this scenario it must be assumed that the bile duct has been damaged. In most cases an ERCP is the most appropriate investigation. This
can also allow the passage of a stent if this is deemed to be safe and sensible.

9. A 34 year old lady is admitted with jaundice and undergoes an ERCP. The procedure is technically difficult and she is returned to the ward
still jaundiced. Unfortunately she now has severe generalised abdominal pain.

You answered Laparotomy and formation of hepatico-jejunostomy

The correct answer is CT scan

There are two main differential diagnoses here. One is pancreatitis, repeated trauma to the ampulla and duct (if partially cannulated) is a
major risk factor for pancreatitis. The second is the possibility that the duodenum has been perforated. ERCP is performed using a side
viewing endoscope, the manipulation of which can be technically challenging for the inexperienced operator in a patient with abnormal
anatomy. A CT scan is the best investigation to distinguish between these two differential diagnoses.

Next question

Surgical jaundice

Jaundice can present in a manner of different surgical situations. As with all types of jaundice a careful history and examination will often give clues as

http://www.emrcs.com/question/question.php?q=0 1/4
eMRCS 19/07/2015

to the most likely underlying cause. Liver function tests whilst conveying little in the way of information about liver synthetic function, will often
facilitate classification as to whether the jaundice is pre hepatic, hepatic or post hepatic. The typical LFT patterns are given below:

Location Bilirubin ALT/ AST Alkaline phosphatase

Pre hepatic Normal or high Normal Normal

Hepatic High Elevated (often very high) Elevated but seldom to very high levels

Post hepatic High-very high Moderate elevation High- very high

In post hepatic jaundice the stools are often of pale colour and this feature should be specifically addressed in the history.

Modes of presentation
These are addressed in the table below:

Diagnosis Typical features Pathogenesis

Gallstones Typically history of biliary colic Usually small calibre gallstones which can pass through the cystic duct. In Mirizzi
or episodes of chlolecystitis. syndrome the stone may compress the bile duct directly- one of the rare times that
Obstructive type history and cholecystitis may present with jaundice
test results.

Cholangitis Usually obstructive and will have Ascending infection of the bile ducts usually by E. coli and by definition occurring in a
Charcots triad of symptoms pool of stagnant bile.
(pain, fever, jaundice)

Pancreatic cancer Typically painless jaundice with Direct occlusion of distal bile duct or pancreatic duct by tumour. Sometimes nodal
palpable gallbladder disease at the portal hepatis may be the culprit in which case the bile duct may be of
(Courvoisier's Law) normal calibre.

TPN associated Usually follows long term use Often due to hepatic dysfunction and fatty liver which may occur with long term TPN
jaundice and is usually painless with non usage.
obstructive features

Bile duct injury Depending upon the type of Often due to a difficult cholecystectomy when anatomy in Calots triangle is not
injury may be of sudden or appreciated. In the worst scenario the bile duct is excised and jaundice offers rapidly
gradual onset and is usually of post operatively. More insidious is that of bile duct stenosis which may be caused by
obstructive type clips or diathermy injury.

Cholangiocarcinoma Gradual onset obstructive Direct occlusion by disease and also extrinsic compression by nodal disease at the porta
pattern hepatis.

Septic surgical Usually hepatic features Combination of impaired biliary excretion and drugs such as ciprofloxacin which may
patient cause cholestasis.

Metastatic disease Mixed hepatic and post hepatic Combination of liver synthetic failure (late) and extrinsic compression by nodal disease
and anatomical compression of intra hepatic structures (earlier)

Diagnosis
An ultrasound of the liver and biliary tree is the most commonly used first line test. This will establish bile duct calibre, often ascertain the presence of
gallstones, may visualise pancreatic masses and other lesions. The most important clinical question is essentially the extent of biliary dilatation and its
distribution.

Where pancreatic neoplasia is suspected, the next test should be a pancreatic protocol CT scan. With liver tumours and cholangiocarcinoma an
MRI/ MRCP is often the preferred option. PET scans may be used to stage a number of malignancies but do not routinely form part of first line
testing.

Where MRCP fails to give adequate information an ERCP may be necessary. In many cases this may form part of patient management. It is
however, invasive and certainly not without risk and highly operator dependent.

Management
Clearly this will depend to an extent upon the underlying cause but relief of jaundice is important, even if surgery forms part of the planned
treatment. Patients with unrelieved jaundice have a much higher incidence of septic complications, bleeding and death.

Screen for and address any clotting irregularities

In patients with malignancy a stent will need to be inserted. These come in two main types; metal and plastic. Plastic stents are cheap and easy to

http://www.emrcs.com/question/question.php?q=0 2/4
eMRCS 19/07/2015

replace and should be used if any surgical intervention (e.g. Whipples) is planned. However, they are prone to displacement and blockage. Metal
stents are much more expensive and may compromise a surgical resection. However, they are far less prone to displacement and to a lesser extent
blockage than their plastic counterparts.

If malignancy is in bile duct/ pancreatic head and stenting has been attempted and has failed, then an alternative strategy is to drain the biliary
system percutaneously via a transhepatic route. It may also be possible to insert a stent in this way. One of the main problems with temporary
PTC's is their propensity to displacement, which may result in a bile leak.

In patients who have a bile duct injury surgery will be required to repair the defect. If the bile duct has been inadvertently excised then a hepatico-
jejunostomy will need to be created (difficult!)

If gallstones are the culprit, then these may be removed by ERCP and a cholecystectomy performed. Where there is doubt about the efficacy of
the ERCP an operative cholangiogram should be performed and bile duct exploration undertaken where stones remain. When the bile duct has been
formally opened the options are between closure over a T tube, a choledochoduodenostomy or choledochojejunostomy.

Patients with cholangitis should receive high dose broad spectrum antibiotics via the intravenous route. Biliary decompression should follow soon
afterwards, instrumenting the bile duct of these patients will often provoke a septic episode (but should be done anyway).

Save my notes

Please rate this question: Next question

Send feedback to the author

Question stats

Average score for registered users:

7 43.4%
8 43.1%
9 46.7%

Search eMRCS

Search term Go

External links

http://www.emrcs.com/question/question.php?q=0 3/4
eMRCS 19/07/2015

Suggest a link

Dashboard

Overall score: 33.3%


1-3 0 / 3
4-6 3/3
7-9 1/3

All contents of this site are © 2015 E-Medical Revision Ltd Back to top

http://www.emrcs.com/question/question.php?q=0 4/4
eMRCS 19/07/2015

0/3
Question 10-12 of 41

Theme: Gallstone disease

A. Uncomplicated biliary colic


B. Acute cholecystitis
C. Cholangitis
D. Gallbladder abscess
E. Acalculous cholecystitis
F. Pancreatitis
G. Gallstone ileus

Please select the most likely underlying diagnosis for the scenario given. Each option may be used once, more than once or not at all.

10. A 68 year old man with type 2 diabetes is admitted to hospital unwell. On examination he has features of septic shock and right upper
quadrant tenderness. He is not jaundiced. Imaging shows a normal calibre bile duct and no stones in the gallbladder.

You answered Pancreatitis

The correct answer is Acalculous cholecystitis

Theme from April 2013 Exam


Acalculous cholecystitis is more common in patients with an underlying co-morbidity. The morbidity and mortality following intervention
are higher than in conventional gallstone disease.

11. A 43 year old lady with known gallstones is admitted with a high fever and jaundice. On examination, she looks extremely unwell. Her
abdomen is generally soft although there is some mild tenderness in the right upper quadrant.

You answered Acute cholecystitis

The correct answer is Cholangitis

Features of jaundice, fever and systemic sepsis are typical of cholangitis.

12. A 34 year old lady is admitted with a 3 day history of colicky right upper quadrant pain which radiates to her back. The pain is now more
constant. On examination she is not jaundiced, but has a temperature of 38.5oC. She has localised peritonism in the right upper quadrant.

You answered Cholangitis

The correct answer is Acute cholecystitis

The features of pain and fever with right upper quadrant pain are suggestive of acute cholecystitis. The short nature of the history makes
an abscess less likely.

Next question

Gallstones

Up to 24% of women and 12% of men may have gallstones. Of these up to 30% may develop local infection and cholecystitis. In patients
subjected to surgery 12% will have stones contained within the common bile duct. The majority of gallstones are of a mixed composition (50%)
with pure cholesterol stones accounting for 20% of cases.
The aetiology of CBD stones differs in the world, in the West most CBD stones are the result of migration. In the East a far higher proportion arise in
the CBD de novo.
The classical symptoms are of colicky right upper quadrant pain that occurs post prandially. The symptoms are usually worst following a fatty meal
when cholecystokinin levels are highest and gallbladder contraction is maximal.

http://www.emrcs.com/question/question.php?q=0 1/4
eMRCS 19/07/2015

Investigation
In almost all suspected cases the standard diagnostic work up consists of abdominal ultrasound and liver function tests. Of patients who have stones
within the bile duct, 60% will have at least one abnormal result on LFT's. Ultrasound is an important test, but is operator dependent and therefore
may occasionally need to be repeated if a negative result is at odds with the clinical picture. Where stones are suspected in the bile duct, the options
lie between magnetic resonance cholangiography and intraoperative imaging. The choice between these two options is determined by the skills and
experience of the surgeon. The advantages of intra operative imaging are less useful in making therapeutic decisions if the operator is unhappy
about proceeding the bile duct exploration, and in such circumstances pre operative MRCP is probably a better option.

Specific gallstone and gallbladder related disease


Disease Features Management

Biliary colic Colicky abdominal pain, worse If imaging shows gallstones and history compatible then laparoscopic cholecystectomy
post prandially, worse after
fatty foods

Acute Right upper quadrant Imaging (USS) and cholecystectomy (ideally within 48 hours of presentation) (2)
cholecystitis pain
Fever
Murphys sign on
examination
Occasionally mildly
deranged LFT's
(especially if Mirizzi
syndrome)

Gallbladder Usually prodromal illness Imaging with USS +/- CT Scanning


abscess and right upper Ideally surgery, sub total cholecystectomy may be needed if Calots triangle is hostile
quadrant pain In unfit patients percutaneous drainage may be considered
Swinging pyrexia
Patient may be
systemically unwell
Generalised peritonism
not present

Cholangitis Patient severely septic Fluid resuscitation


and unwell Broad spectrum intravenous antibiotics
Jaundice Correct any coagulopathy
Right upper quadrant Early ERCP
pain

Gallstone Patients may have a Laparotomy and removal of gallstone from small bowel, the enterotomy must be made proximal
ileus history of previous to the site of obstruction and not at the site of obstruction. The fistula between the gallbladder
cholecystitis and known and duodenum should not be interfered with.
gallstones
Small bowel obstruction
(may be intermittent)

Acalculous Patients with inter If patient fit then cholecystectomy, if unfit then percutaneous cholecystostomy
cholecystitis current illness (e.g.
diabetes, organ failure)
Patient of systemically
unwell
Gallbladder inflammation
in absence of stones
High fever

Treatment
Patients with asymptomatic gallstones rarely develop symptoms related to them (less than 2% per year) and may therefore be managed
expectantly. In almost all cases of symptomatic gallstones the treatment of choice is cholecystectomy performed via the laparoscopic route. In the
very frail patient there is sometimes a role for selective use of ultrasound guided cholecystostomy.
During the course of the procedure some surgeons will routinely perform either intra operative cholangiography to either confirm anatomy or to
exclude CBD stones. The latter may be more easily achieved by use of laparoscopic ultrasound. If stones are found then the options lie between
early ERCP in the day or so following surgery or immediate surgical exploration of the bile duct. When performed via the trans cystic route this adds
little in the way of morbidity and certainly results in faster recovery. Where transcystic exploration fails the alternative strategy is that of formal
choledochotomy. The exploration of a small duct is challenging and ducts of less than 8mm should not be explored. Small stones that measure less
than 5mm may be safely left and most will pass spontaneously.

http://www.emrcs.com/question/question.php?q=0 2/4
eMRCS 19/07/2015

Risks of ERCP(1)
Bleeding 0.9% (rises to 1.5% if sphincterotomy performed)
Duodenal perforation 0.4%
Cholangitis 1.1%
Pancreatitis 1.5%

References
1. Williams E et al. Guidelines on the management of common bile duct stones (CBDS)Gut 2008;57:10041021

2. Gurusamy KS, Samraj K. Early versus delayed laparoscopic cholecystectomy for acute cholecystitis. Cochrane Database Syst Rev. 2006 Oct
18;(4):CD005440.

3. Gurusamy K and Davidson B. Gallstones. BMJ 2014 (348):27-30.

Save my notes

Please rate this question: Next question

Send feedback to the author

Question stats

Average score for registered users:

10 55.2%
11 70.5%
12 53%

Search eMRCS

Search term Go

External links

Suggest a link

http://www.emrcs.com/question/question.php?q=0 3/4
eMRCS 19/07/2015

Dashboard

Overall score: 26.7%


1-3 0 / 3
4-6 3/3
7-9 1/3
10-
0/3
12

All contents of this site are © 2015 E-Medical Revision Ltd Back to top

http://www.emrcs.com/question/question.php?q=0 4/4
eMRCS 19/07/2015

Question 13 of 41

Which of the following is the most sensitive blood test for diagnosis of acute pancreatitis?

Amylase

Lipase

C-peptide

Trypsin

Trysinogen

Next question

The serum amylase may rise and fall quite quickly and lead to a false negative result. Should the clinical picture not be concordant with the amylase
level then serum lipase or a CT Scan should be performed.

Management of Pancreatitis

Management of Acute Pancreatitis in the UK

Diagnosis
Traditionally hyperamylasaemia has been utilised with amylase being elevated three times the normal range.
However, amylase may give both false positive and negative results.
Serum lipase is both more sensitive and specific than serum amylase. It also has a longer half life.
Serum amylase levels do not correlate with disease severity.

Differential causes of hyperamylasaemia


Acute pancreatitis

Pancreatic pseudocyst

Mesenteric infarct

Perforated viscus

Acute cholecystitis

Diabetic ketoacidosis

Assessment of severity
Glasgow, Ranson scoring systems and APACHE II
Biochemical scoring e.g. using CRP

Features that may predict a severe attack within 48 hours of admission to hospital
Initial assessment Clinical impression of severity
Body mass index >30
Pleural effusion
APACHE score >8

24 hours after admission Clinical impression of severity


APACHE II >8

http://www.emrcs.com/question/question.php?q=0 1/3
eMRCS 19/07/2015

Glasgow score of 3 or more


Persisting multiple organ failure
CRP>150

48 hours after admission Glasgow Score of >3


CRP >150
Persisting or progressive organ failure

Table adapted from UK guidelines for management of acute pancreatitis. GUT 2005, 54 suppl III

Management

Nutrition
There is reasonable evidence to suggest that the use of enteral nutrition does not worsen the outcome in pancreatitis
Most trials to date were underpowered to demonstrate a conclusive benefit.
The rationale behind feeding is that it helps to prevent bacterial translocation from the gut, thereby contributing to the development of
infected pancreatic necrosis.

Use of antibiotic therapy


Many UK surgeons administer antibiotics to patients with acute pancreatitis. However, there is very little evidence to support this practice.
A recent Cochrane review highlights the potential benefits of administering Imipenem to patients with established pancreatic necrosis in the
hope of averting the progression to infection.
There are concerns that the administration of antibiotics in mild attacks of pancreatitis will not affect outcome and may contribute to antibiotic
resistance and increase the risks of antibiotic associated diarrhoea.

Surgery
Patients with acute pancreatitis due to gallstones should undergo early cholecystectomy.
Patients with obstructed biliary system due to stones should undergo early ERCP.
Patients with extensive necrosis where infection is suspected should usually undergo FNA for culture.
Patients with infected necrosis should undergo either radiological drainage or surgical necrosectomy. The choice of procedure depends upon
local expertise.

References
www.bsg.org.uk/pdfworddocs/pancreatic.pdf

Antibiotic therapy for prophylaxis against infection of pancreatic necrosis in acute pancreatitis. Villatoro et al. Cochrane Library DOI:
10.1002/14651858.CD002941.pub3. 2010 version.

Save my notes

Please rate this question: Next question

Send feedback to the author

http://www.emrcs.com/question/question.php?q=0 2/3
eMRCS 19/07/2015

Question stats

A 27.3%
B 49.7%
C 8.5%
D 7.3%
E 7.2%

49.7% of users answered this question correctly

Search eMRCS

Search term Go

External links

Suggest a link

Dashboard

Overall score: 25%


1-3 0 / 3
4-6 3/3
7-9 1/3
10-12 0 / 3
13

All contents of this site are © 2015 E-Medical Revision Ltd Back to top

http://www.emrcs.com/question/question.php?q=0 3/3
eMRCS 19/07/2015

0/3
Question 4-6 of 962

Theme: Jaundice

A. Gilberts syndrome
B. Crigler Najjar syndrome
C. Hepatocellular carcinoma
D. Mirizzi syndrome
E. Hepatitis A
F. Hepatitis E
G. Bile duct stones
H. Multi cystic liver disease

Please select the most likely cause of jaundice for the scenario given. Each option may be used once, more than once or not at all.

4. A 22 year old man returns to the UK from holiday in India. He presents with painless jaundice. On examination he is not deeply jaundiced
and there is no organomegaly.

You answered Hepatocellular carcinoma

The correct answer is Hepatitis A

Infective hepatitis is the most likely cause. In the UK, foreign travel is a common cause of developing infectious hepatitis, of which hepatitis A
is the most common.

5. A 56 year old man presents with jaundice. He has a long history of alcohol misuse. On examination he is jaundiced and ultrasound shows
multiple echo dense lesions in both lobes of the liver. His alpha feto protein is elevated 6 times the normal range

You answered Hepatitis E

The correct answer is Hepatocellular carcinoma

HCC may complicate cirrhosis. AFP is often raised in HCC.

6. A 32 year old man who has suffered from Crohns disease for many years presents with intermittent jaundice. When it occurs it is
obstructive in nature. It then usually resolves spontaneously.

You answered Gilberts syndrome

The correct answer is Bile duct stones

Bile salts are absorbed in the terminal ileum. When this process is impaired as in Crohns the patient may develop gallstones, if these pass into
the CBD then obstructive jaundice will result.

Next question

Surgical jaundice

Jaundice can present in a manner of different surgical situations. As with all types of jaundice a careful history and examination will often give clues as
to the most likely underlying cause. Liver function tests whilst conveying little in the way of information about liver synthetic function, will often
facilitate classification as to whether the jaundice is pre hepatic, hepatic or post hepatic. The typical LFT patterns are given below:

Location Bilirubin ALT/ AST Alkaline phosphatase

http://www.emrcs.com/question/question.php?q=1 1 / 22
eMRCS 19/07/2015

Pre hepatic Normal or high Normal Normal

Hepatic High Elevated (often very high) Elevated but seldom to very high levels

Post hepatic High-very high Moderate elevation High- very high

In post hepatic jaundice the stools are often of pale colour and this feature should be specifically addressed in the history.

Modes of presentation
These are addressed in the table below:

Diagnosis Typical features Pathogenesis

Gallstones Typically history of biliary colic Usually small calibre gallstones which can pass through the cystic duct. In Mirizzi
or episodes of chlolecystitis. syndrome the stone may compress the bile duct directly- one of the rare times that
Obstructive type history and cholecystitis may present with jaundice
test results.

Cholangitis Usually obstructive and will have Ascending infection of the bile ducts usually by E. coli and by definition occurring in a
Charcots triad of symptoms pool of stagnant bile.
(pain, fever, jaundice)

Pancreatic cancer Typically painless jaundice with Direct occlusion of distal bile duct or pancreatic duct by tumour. Sometimes nodal
palpable gallbladder disease at the portal hepatis may be the culprit in which case the bile duct may be of
(Courvoisier's Law) normal calibre.

TPN associated Usually follows long term use Often due to hepatic dysfunction and fatty liver which may occur with long term TPN
jaundice and is usually painless with non usage.
obstructive features

Bile duct injury Depending upon the type of Often due to a difficult cholecystectomy when anatomy in Calots triangle is not
injury may be of sudden or appreciated. In the worst scenario the bile duct is excised and jaundice offers rapidly
gradual onset and is usually of post operatively. More insidious is that of bile duct stenosis which may be caused by
obstructive type clips or diathermy injury.

Cholangiocarcinoma Gradual onset obstructive Direct occlusion by disease and also extrinsic compression by nodal disease at the porta
pattern hepatis.

Septic surgical Usually hepatic features Combination of impaired biliary excretion and drugs such as ciprofloxacin which may
patient cause cholestasis.

Metastatic disease Mixed hepatic and post hepatic Combination of liver synthetic failure (late) and extrinsic compression by nodal disease
and anatomical compression of intra hepatic structures (earlier)

Diagnosis
An ultrasound of the liver and biliary tree is the most commonly used first line test. This will establish bile duct calibre, often ascertain the presence of
gallstones, may visualise pancreatic masses and other lesions. The most important clinical question is essentially the extent of biliary dilatation and its
distribution.

Where pancreatic neoplasia is suspected, the next test should be a pancreatic protocol CT scan. With liver tumours and cholangiocarcinoma an
MRI/ MRCP is often the preferred option. PET scans may be used to stage a number of malignancies but do not routinely form part of first line
testing.

Where MRCP fails to give adequate information an ERCP may be necessary. In many cases this may form part of patient management. It is
however, invasive and certainly not without risk and highly operator dependent.

Management
Clearly this will depend to an extent upon the underlying cause but relief of jaundice is important, even if surgery forms part of the planned
treatment. Patients with unrelieved jaundice have a much higher incidence of septic complications, bleeding and death.

Screen for and address any clotting irregularities

In patients with malignancy a stent will need to be inserted. These come in two main types; metal and plastic. Plastic stents are cheap and easy to
replace and should be used if any surgical intervention (e.g. Whipples) is planned. However, they are prone to displacement and blockage. Metal
stents are much more expensive and may compromise a surgical resection. However, they are far less prone to displacement and to a lesser extent
blockage than their plastic counterparts.

If malignancy is in bile duct/ pancreatic head and stenting has been attempted and has failed, then an alternative strategy is to drain the biliary
system percutaneously via a transhepatic route. It may also be possible to insert a stent in this way. One of the main problems with temporary

http://www.emrcs.com/question/question.php?q=1 2 / 22
eMRCS 19/07/2015

PTC's is their propensity to displacement, which may result in a bile leak.

In patients who have a bile duct injury surgery will be required to repair the defect. If the bile duct has been inadvertently excised then a hepatico-
jejunostomy will need to be created (difficult!)

If gallstones are the culprit, then these may be removed by ERCP and a cholecystectomy performed. Where there is doubt about the efficacy of
the ERCP an operative cholangiogram should be performed and bile duct exploration undertaken where stones remain. When the bile duct has been
formally opened the options are between closure over a T tube, a choledochoduodenostomy or choledochojejunostomy.

Patients with cholangitis should receive high dose broad spectrum antibiotics via the intravenous route. Biliary decompression should follow soon
afterwards, instrumenting the bile duct of these patients will often provoke a septic episode (but should be done anyway).

Save my notes

Please rate this question: Next question

Send feedback to the author

Question stats

Average score for registered users:

4 72.4%
5 88.5%
6 65.9%

Search eMRCS

Search term Go

External links

Suggest a link

Dashboard

http://www.emrcs.com/question/question.php?q=1 3 / 22
eMRCS 19/07/2015

1-3 0/3
4-6 0/3
7
8-10 0 / 3
11-13 1 / 3
14-16 3 / 3
17-19 0 / 3
20

21

22
23-25 0 / 3
26
27-29 0 / 3
30-32 1 / 3
33
34-36 1 / 3
37

38

39
40-42 0 / 3
43

44
45-47 0 / 3
48-50 0 / 3
51-53 1 / 3
54
55

56

57

58

59
60

61

62

63

64
65
66-68 0 / 3
69
70-72 1 / 3
73

http://www.emrcs.com/question/question.php?q=1 4 / 22
eMRCS 19/07/2015

74
75-77 2 / 3
78-80 0 / 3
81

82

83
84

85

86

87
88-90 1 / 3
91-93 0 / 3
94
95
96-98 0 / 3
99-
1/3
101
102
103-
0/3
105
106
107-
0/3
109
110-
0/3
112
113
114

115

116
117-
0/3
119
120-
1/3
122
123
124-
0/3
126
127-
0/3
129
130-
1/3
132
133-
0/3
135
136-
1/3
138
139
140
141-
0/3
143

http://www.emrcs.com/question/question.php?q=1 5 / 22
eMRCS 19/07/2015

144-
1/3
146
147

148
149-
0/3
151
152
153-
0/3
155
156

157
158
159-
0/3
161
162

163

164
165-
1/3
167
168

169

170

171

172
173

174

175

176

177
178-
0/3
180
181
182-
1/3
184
185

186

187

188

189

190
191-
0/3
193
194

195
196-
0/3

http://www.emrcs.com/question/question.php?q=1 6 / 22
eMRCS 19/07/2015

0/3
198
199
200-
0/3
202
203
204-
0/3
206
207
208
209-
1/3
211
212

213
214
215-
0/3
217
218

219
220

221

222

223

224

225

226
227-
1/3
229
230

231

232

233

234

235
236

237

238

239
240-
0/3
242
243

244

245

246

247

http://www.emrcs.com/question/question.php?q=1 7 / 22
eMRCS 19/07/2015

248
249-
0/3
251
252-
0/3
254
255
256-
2/3
258
259
260

261
262-
0/3
264
265
266

267

268

269
270-
1/3
272
273

274

275
276-
1/3
278
279
280-
0/3
282
283
284-
0/3
286
287

288

289
290

291

292
293-
1/3
295
296
297-
0/3
299
300-
1/3
302
303

304

http://www.emrcs.com/question/question.php?q=1 8 / 22
eMRCS 19/07/2015

305-
0/3
307
308

309

310

311

312
313

314
315-
0/3
317
318
319-
0/3
321
322-
0/3
324
325

326
327-
1/3
329
330-
1/3
332
333
334-
0/3
336
337
338-
1/3
340
341

342

343
344

345
346-
0/3
348
349-
0/3
351
352

353

354

355
356

357
358-
0/3
360
361-
1/3
363

http://www.emrcs.com/question/question.php?q=1 9 / 22
eMRCS 19/07/2015

364-
1/3
366
367
368-
1/3
370
371
372-
0/3
374
375

376
377-
0/3
379
380-
0/3
382
383-
0/3
385
386-
2/3
388
389-
0/3
391
392-
1/3
394
395

396

397

398

399
400

401
402-
0/3
404
405
406

407

408

409

410
411-
0/3
413
414-
0/3
416
417
418-
1/3
420
421-
0/3
423
424
425-
0/3
427

http://www.emrcs.com/question/question.php?q=1 10 / 22
eMRCS 19/07/2015

428-
1/3
430
431
432-
0/3
434
435

436
437

438

439

440

441

442

443

444

445

446
447
448-
0/3
450
451-
2/3
453
454-
0/3
456
457
458-
1/3
460
461

462

463

464
465-
1/3
467
468
469-
0/3
471
472-
0/3
474
475-
1/3
477
478-
1/3
480
481

482

483

484

http://www.emrcs.com/question/question.php?q=1 11 / 22
eMRCS 19/07/2015

485-
1/3
487
488
489-
0/3
491
492
493-
1/3
495
496-
2/3
498
499-
3/3
501
502
503-
0/3
505
506

507

508

509
510

511

512

513

514

515
516-
0/3
518
519-
1/3
521
522-
1/3
524
525

526

527

528

529

530

531
532-
0/3
534
535

536

537

538

539
540-
0/3

http://www.emrcs.com/question/question.php?q=1 12 / 22
eMRCS 19/07/2015

0/3
542
543
544-
0/3
546
547

548

549

550

551

552
553-
1/3
555
556

557

558
559-
1/3
561
562

563

564

565
566-
0/3
568
569

570

571

572

573
574

575

576

577

578

579
580-
0/3
582
583

584
585-
0/3
587
588
589-
0/3
591
592

http://www.emrcs.com/question/question.php?q=1 13 / 22
eMRCS 19/07/2015

593

594

595

596

597

598
599-
0/3
601
602

603

604

605

606

607

608

609

610

611

612

613

614
615-
0/3
617
618

619

620

621

622

623

624

625

626

627

628

629

630

631

632

633

634
635

http://www.emrcs.com/question/question.php?q=1 14 / 22
eMRCS 19/07/2015

636

637

638

639

640
641-
0/3
643
644

645

646

647

648

649

650
651
652-
0/3
654
655

656

657
658-
0/3
660
661

662
663

664

665

666

667

668
669-
0/3
671
672

673

674

675

676
677-
0/3
679
680

681

682

683

http://www.emrcs.com/question/question.php?q=1 15 / 22
eMRCS 19/07/2015

684

685

686

687

688

689
690-
1/3
692
693

694

695

696

697
698-
1/3
700
701

702

703
704-
1/3
706
707

708

709

710

711

712

713
714-
1/3
716
717

718
719

720

721

722

723

724

725

726

727

728

729

http://www.emrcs.com/question/question.php?q=1 16 / 22
eMRCS 19/07/2015

730-
0/3
732
733

734

735
736

737

738

739

740

741

742

743

744

745

746

747

748

749

750

751

752
753-
0/3
755
756

757

758

759

760

761

762

763

764
765-
1/3
767
768
769

770

771

772

773
774-
0/3

http://www.emrcs.com/question/question.php?q=1 17 / 22
eMRCS 19/07/2015

0/3
776
777

778

779

780

781

782

783

784
785

786

787

788

789

790

791
792-
1/3
794
795

796

797

798

799

800
801

802

803
804-
0/3
806
807

808
809-
0/3
811
812
813

814

815

816

817

818

819

820

http://www.emrcs.com/question/question.php?q=1 18 / 22
eMRCS 19/07/2015

821

822

823
824-
0/3
826
827

828

829
830-
1/3
832
833

834

835

836

837

838

839
840-
1/3
842
843

844

845

846
847-
1/3
849
850

851

852

853

854
855-
0/3
857
858

859

860

861

862

863

864

865

866

867

868

http://www.emrcs.com/question/question.php?q=1 19 / 22
eMRCS 19/07/2015

869
870-
0/3
872
873
874-
0/3
876
877-
0/3
879
880

881
882

883

884

885

886

887

888

889

890

891

892

893

894

895

896

897

898

899

900

901

902
903

904
905-
1/3
907
908

909

910

911

912

913

914

http://www.emrcs.com/question/question.php?q=1 20 / 22
eMRCS 19/07/2015

915

916

917

918
919

920
921-
0/3
923
924

925

926

927

928

929

930

931

932

933

934
935

936

937

938

939
940-
0/3
942
943

944

945
946-
0/3
948
949

950
951-
1/3
953
954

955

956
957-
0/3
959
960

961

962

http://www.emrcs.com/question/question.php?q=1 21 / 22
eMRCS 19/07/2015

963

All contents of this site are © 2015 E-Medical Revision Ltd Back to top

http://www.emrcs.com/question/question.php?q=1 22 / 22
eMRCS 19/07/2015

Question 1 of 22

A 59 year old man is diagnosed as having carcinoma of the pancreas with two malignant deposits in the right lobe of the liver. What is the most
appropriate treatment?

Palliative chemotherapy

Liver resection followed by chemotherapy

Simultaneous resection of liver metastasis and en bloc segmental pancreatic resection

Pancreatic resection followed by liver resection once recovered

Radical radiotherapy followed by surgery

Next question

Pancreatic cancer has a poor prognosis and most cases have metastatic disease at presentation. There is no role in pancreatic cancer for liver
resection together with pancreatic surgery as there is no survival benefit. Most centres will offer palliative chemotherapy which has improved both
longevity and quality of life.

Pancreatic cancer

Adenocarcinoma
Risk factors: Smoking, diabetes, Adenoma, Familial adenomatous polyposis
Mainly occur in the head of the pancreas (70%)
Spread locally and metastasizes to the liver
Carcinoma of the pancreas should be differentiated from other periampullary tumours with better prognosis

Clinical features
Weight loss
Painless jaundice
Epigastric discomfort (pain usually due to invasion of the coeliac plexus is a late feature)
Pancreatitis
Trousseau's sign: migratory superficial thrombophlebitis

Investigations
USS: May miss small lesions
CT Scanning (pancreatic protocol). If unresectable on CT then no further staging needed
PET/CT for those with operable disease on CT alone
ERCP/ MRI for bile duct assessment
Staging laparoscopy to exclude peritoneal disease

Management
Head of pancreas: Whipple's resection (SE dumping and ulcers). Newer techniques include pylorus preservation and SMA/ SMV resection
Carcinoma body and tail: poor prognosis, distal pancreatectomy, if operable
Usually adjuvent chemotherapy for resectable disease
ERCP and stent for jaundice and palliation
Surgical bypass may be needed for duodenal obstruction

http://www.emrcs.com/question/question.php?q=0 1/2
eMRCS 19/07/2015

Save my notes

Please rate this question: Next question

Send feedback to the author

Question stats

A 48.9%
B 7.2%
C 20.9%
D 11%
E 12%

48.9% of users answered this question correctly

Search eMRCS

Search term Go

External links

Suggest a link

Dashboard

Overall score: 0%
1

All contents of this site are © 2015 E-Medical Revision Ltd Back to top

http://www.emrcs.com/question/question.php?q=0 2/2
eMRCS 19/07/2015

Question 2 of 22

A 41 year old lady with colicky right upper quadrant pain is identified as having gallstones on an abdominal ultrasound scan. What is the most
appropriate course of action?

Laparoscopic cholecystectomy

Open cholecystectomy

Liver function tests

MRCP

ERCP

Next question

Liver function testing is part of the core diagnostic work up of biliary colic and surgical planning cannot proceed until this (and the diameter of the
CBD on USS) are known.

Gallstones

Up to 24% of women and 12% of men may have gallstones. Of these up to 30% may develop local infection and cholecystitis. In patients
subjected to surgery 12% will have stones contained within the common bile duct. The majority of gallstones are of a mixed composition (50%)
with pure cholesterol stones accounting for 20% of cases.
The aetiology of CBD stones differs in the world, in the West most CBD stones are the result of migration. In the East a far higher proportion arise in
the CBD de novo.
The classical symptoms are of colicky right upper quadrant pain that occurs post prandially. The symptoms are usually worst following a fatty meal
when cholecystokinin levels are highest and gallbladder contraction is maximal.

Investigation
In almost all suspected cases the standard diagnostic work up consists of abdominal ultrasound and liver function tests. Of patients who have stones
within the bile duct, 60% will have at least one abnormal result on LFT's. Ultrasound is an important test, but is operator dependent and therefore
may occasionally need to be repeated if a negative result is at odds with the clinical picture. Where stones are suspected in the bile duct, the options
lie between magnetic resonance cholangiography and intraoperative imaging. The choice between these two options is determined by the skills and
experience of the surgeon. The advantages of intra operative imaging are less useful in making therapeutic decisions if the operator is unhappy
about proceeding the bile duct exploration, and in such circumstances pre operative MRCP is probably a better option.

Specific gallstone and gallbladder related disease


Disease Features Management

Biliary colic Colicky abdominal pain, worse If imaging shows gallstones and history compatible then laparoscopic cholecystectomy
post prandially, worse after
fatty foods

Acute Right upper quadrant Imaging (USS) and cholecystectomy (ideally within 48 hours of presentation) (2)
cholecystitis pain
Fever
Murphys sign on
examination
Occasionally mildly
deranged LFT's
(especially if Mirizzi
syndrome)

Gallbladder Usually prodromal illness Imaging with USS +/- CT Scanning


abscess and right upper Ideally surgery, sub total cholecystectomy may be needed if Calots triangle is hostile

http://www.emrcs.com/question/question.php?q=0 1/3
eMRCS 19/07/2015

quadrant pain In unfit patients percutaneous drainage may be considered


Swinging pyrexia
Patient may be
systemically unwell
Generalised peritonism
not present

Cholangitis Patient severely septic Fluid resuscitation


and unwell Broad spectrum intravenous antibiotics
Jaundice Correct any coagulopathy
Right upper quadrant Early ERCP
pain

Gallstone Patients may have a Laparotomy and removal of gallstone from small bowel, the enterotomy must be made proximal
ileus history of previous to the site of obstruction and not at the site of obstruction. The fistula between the gallbladder
cholecystitis and known and duodenum should not be interfered with.
gallstones
Small bowel obstruction
(may be intermittent)

Acalculous Patients with inter If patient fit then cholecystectomy, if unfit then percutaneous cholecystostomy
cholecystitis current illness (e.g.
diabetes, organ failure)
Patient of systemically
unwell
Gallbladder inflammation
in absence of stones
High fever

Treatment
Patients with asymptomatic gallstones rarely develop symptoms related to them (less than 2% per year) and may therefore be managed
expectantly. In almost all cases of symptomatic gallstones the treatment of choice is cholecystectomy performed via the laparoscopic route. In the
very frail patient there is sometimes a role for selective use of ultrasound guided cholecystostomy.
During the course of the procedure some surgeons will routinely perform either intra operative cholangiography to either confirm anatomy or to
exclude CBD stones. The latter may be more easily achieved by use of laparoscopic ultrasound. If stones are found then the options lie between
early ERCP in the day or so following surgery or immediate surgical exploration of the bile duct. When performed via the trans cystic route this adds
little in the way of morbidity and certainly results in faster recovery. Where transcystic exploration fails the alternative strategy is that of formal
choledochotomy. The exploration of a small duct is challenging and ducts of less than 8mm should not be explored. Small stones that measure less
than 5mm may be safely left and most will pass spontaneously.

Risks of ERCP(1)
Bleeding 0.9% (rises to 1.5% if sphincterotomy performed)
Duodenal perforation 0.4%
Cholangitis 1.1%
Pancreatitis 1.5%

References
1. Williams E et al. Guidelines on the management of common bile duct stones (CBDS)Gut 2008;57:10041021

2. Gurusamy KS, Samraj K. Early versus delayed laparoscopic cholecystectomy for acute cholecystitis. Cochrane Database Syst Rev. 2006 Oct
18;(4):CD005440.

3. Gurusamy K and Davidson B. Gallstones. BMJ 2014 (348):27-30.

Save my notes

http://www.emrcs.com/question/question.php?q=0 2/3
eMRCS 19/07/2015

Please rate this question: Next question

Send feedback to the author

Question stats

A 34.4%
B 6.3%
C 34.7%
D 10.2%
E 14.4%

34.7% of users answered this question correctly

Search eMRCS

Search term Go

External links

Suggest a link

Dashboard

Overall score: 0%
1
2

All contents of this site are © 2015 E-Medical Revision Ltd Back to top

http://www.emrcs.com/question/question.php?q=0 3/3
eMRCS 19/07/2015

Question 3 of 22

A 42 year old female presents with symptoms of biliary colic and on investigation is identified as having gallstones. Of the procedures listed below,
which is most likely to increase the risk of gallstone formation?

Partial gastrectomy

Jejunal resection

Liver lobectomy

Ileal resection

Left hemicolectomy

Next question

Bile salt reabsorption occurs at the ileum. Therefore cholesterol gallstones form as a result of ileal resection.

Biliary disease

Diagnosis Typical features Pathogenesis

Gallstones Typically history of biliary colic Usually small calibre gallstones which can pass through the cystic duct. In Mirizzi
or episodes of chlolecystitis. syndrome the stone may compress the bile duct directly- one of the rare times that
Obstructive type history and cholecystitis may present with jaundice
test results.

Cholangitis Usually obstructive and will Ascending infection of the bile ducts usually by E. coli and by definition occurring in a
have Charcot's triad of pool of stagnant bile.
symptoms (pain, fever,
jaundice)

Pancreatic cancer Typically painless jaundice with Direct occlusion of distal bile duct or pancreatic duct by tumour. Sometimes nodal
palpable gallbladder disease at the portal hepatis may be the culprit in which case the bile duct may be of
(Courvoisier's Law) normal calibre.

TPN (total Usually follows long term use Often due to hepatic dysfunction and fatty liver which may occur with long term TPN
parenteral nutrition) and is usually painless with non usage.
associated jaundice obstructive features

Bile duct injury Depending upon the type of Often due to a difficult laparoscopic​ cholecystectomy when anatomy in Calots triangle is
injury may be of sudden or not appreciated. In the worst scenario the bile duct is excised and jaundice develops
gradual onset and is usually of rapidly post operatively. More insidious is that of bile duct stenosis which may be caused
obstructive type by clips or diathermy injury.

Cholangiocarcinoma Gradual onset obstructive Direct occlusion by disease and also extrinsic compression by nodal disease at the porta
pattern hepatis.

Septic surgical Usually hepatic features Combination of impaired biliary excretion and drugs such as ciprofloxacin which may
patient cause cholestasis.

Metastatic disease Mixed hepatic and post Combination of liver synthetic failure (late) and extrinsic compression by nodal disease
hepatic and anatomical compression of intra hepatic structures (earlier)

A gallbladder may develop a thickened wall in chronic cholecystitis, microscopically Roikitansky-Aschoff Sinuses may be seen

http://www.emrcs.com/question/question.php?q=0 1/3
eMRCS 19/07/2015

Image sourced from Wikipedia

Save my notes

Please rate this question: Next question

Send feedback to the author

Question stats

A 14.7%
B 13.6%
C 8.1%
D 54.9%
E 8.7%

54.9% of users answered this question correctly

Search eMRCS

Search term Go

http://www.emrcs.com/question/question.php?q=0 2/3
eMRCS 19/07/2015

External links

Suggest a link

Dashboard

Overall score: 0%
1
2

All contents of this site are © 2015 E-Medical Revision Ltd Back to top

http://www.emrcs.com/question/question.php?q=0 3/3
eMRCS 19/07/2015

0/3
Question 4-6 of 22

Theme: Management of pancreatitis

A. Non Contrast enhanced CT scan


B. USS abdomen
C. ERCP alone
D. ERCP with Sphincterotomy and biliary drainage
E. Fine needle aspiration of necrosis
F. Pancreatic necrosectomy
G. Contrast enhanced CT scan

What is the next best step in management for the scenario given? Each option may be used once, more than once or not at all.

4. A 58 year old woman is admitted with an attack of severe acute pancreatitis. She is managed on the intensive care unit and is making
progress. She then deteriorates and a CT scan shows extensive pancreatic necrosis (>40%). There are concerns that this may have
become infected.

You answered USS abdomen

The correct answer is Fine needle aspiration of necrosis

When there are concerns that pancreatic necrosis may have become infected the usual approach is to perform an image guided FNA for
culture. There is always the risk of seeding infection with such a strategy so it must be performed with care. Pancreatic necrosectomy is not
usually undertaken until the presence of infection is proven.

5. A 22 year old teacher is admitted with severe epigastric pain. Serum amylase is normal. You wish to exclude a perforated viscus, and
determine whether pancreatitis is present.

You answered ERCP with Sphincterotomy and biliary drainage

The correct answer is Contrast enhanced CT scan

An ultrasound will not accurately answer this question. Therefore a CT scan is required. Oral and IV contrast would usually be given.

6. A 55 year old accountant has jaundice and a temperature of 39oC. He is known to have gallstones. Blood cultures have grown a gram
negative bacilli. Imaging shows a bile duct measuring 1.2cm in diameter.

You answered Fine needle aspiration of necrosis

The correct answer is ERCP with Sphincterotomy and biliary drainage

You should suspect cholangitis in a patient with fevers and jaundice. Charcot's triad may only be present in 20% of patients. This patient
needs biliary drainage with an ERCP.

Next question

Infected pancreatic necrosis is one of the few indications for surgery in pancreatitis

Management of Pancreatitis

Management of Acute Pancreatitis in the UK

Diagnosis
Traditionally hyperamylasaemia has been utilised with amylase being elevated three times the normal range.
However, amylase may give both false positive and negative results.

http://www.emrcs.com/question/question.php?q=0 1/4
eMRCS 19/07/2015

Serum lipase is both more sensitive and specific than serum amylase. It also has a longer half life.
Serum amylase levels do not correlate with disease severity.

Differential causes of hyperamylasaemia


Acute pancreatitis

Pancreatic pseudocyst

Mesenteric infarct

Perforated viscus

Acute cholecystitis

Diabetic ketoacidosis

Assessment of severity
Glasgow, Ranson scoring systems and APACHE II
Biochemical scoring e.g. using CRP

Features that may predict a severe attack within 48 hours of admission to hospital
Initial assessment Clinical impression of severity
Body mass index >30
Pleural effusion
APACHE score >8

24 hours after admission Clinical impression of severity


APACHE II >8
Glasgow score of 3 or more
Persisting multiple organ failure
CRP>150

48 hours after admission Glasgow Score of >3


CRP >150
Persisting or progressive organ failure

Table adapted from UK guidelines for management of acute pancreatitis. GUT 2005, 54 suppl III

Management

Nutrition
There is reasonable evidence to suggest that the use of enteral nutrition does not worsen the outcome in pancreatitis
Most trials to date were underpowered to demonstrate a conclusive benefit.
The rationale behind feeding is that it helps to prevent bacterial translocation from the gut, thereby contributing to the development of
infected pancreatic necrosis.

Use of antibiotic therapy


Many UK surgeons administer antibiotics to patients with acute pancreatitis. However, there is very little evidence to support this practice.
A recent Cochrane review highlights the potential benefits of administering Imipenem to patients with established pancreatic necrosis in the
hope of averting the progression to infection.
There are concerns that the administration of antibiotics in mild attacks of pancreatitis will not affect outcome and may contribute to antibiotic
resistance and increase the risks of antibiotic associated diarrhoea.

Surgery
Patients with acute pancreatitis due to gallstones should undergo early cholecystectomy.
Patients with obstructed biliary system due to stones should undergo early ERCP.
Patients with extensive necrosis where infection is suspected should usually undergo FNA for culture.
Patients with infected necrosis should undergo either radiological drainage or surgical necrosectomy. The choice of procedure depends upon
local expertise.

References
www.bsg.org.uk/pdfworddocs/pancreatic.pdf

Antibiotic therapy for prophylaxis against infection of pancreatic necrosis in acute pancreatitis. Villatoro et al. Cochrane Library DOI:
10.1002/14651858.CD002941.pub3. 2010 version.

http://www.emrcs.com/question/question.php?q=0 2/4
eMRCS 19/07/2015

Save my notes

Please rate this question: Next question

Send feedback to the author

Question stats

Average score for registered users:

4 62.9%
5 63%
6 77.9%

Search eMRCS

Search term Go

External links

Suggest a link

Dashboard

Overall score: 0%
1

3
4-6 0/3

http://www.emrcs.com/question/question.php?q=0 3/4
eMRCS 19/07/2015

All contents of this site are © 2015 E-Medical Revision Ltd Back to top

http://www.emrcs.com/question/question.php?q=0 4/4
eMRCS 19/07/2015

Question 7 of 22

A 43 year old male with long standing chronic hepatitis is being followed up. Recently his AFP is noted to be increased and an abdominal USS
demonstrates a 2cm lesion in segment V of the liver. What is the most appropriate course of action?

PET CT scan

Liver MRI

USS guided liver biopsy

Laparoscopic biopsy

Segmental resection of segment V

Next question

Liver lesions that are suspicious of HCC should be scanned prior to resection as there is a risk of multifocal lesions that would either preclude or
otherwise affect the decision to proceed with segmental resection.

Hepatocellular carcinoma

Hepatocellular carcinoma is the second leading cause of cancer deaths globally. Up to 750,000 cases are reported annually. Unfortunately the
incidence approximates to the death rate so there are few long term survivors[1]. The disease occurs most commonly in those with chronic
hepatitis and established liver cirrhosis. Therefore, these individuals should be closely screened for the development of HCC with serum AFP and liver
USS every 6-12 months. Rising AFP and liver USS showing a nodule greater than 1cm in diameter makes HCC much more likely and such patients
should then undergo MRI scanning.
The presence of adenomas in an otherwise healthy liver is a recognised risk factor for HCC [2, 3] and many surgeons will remove liver adenomas for
this reason[4].

Diagnosis
The aim is to avoid unnecessary percutaneous biopsy. Radiologically on CT the classical feature is a suspicious lesion which is highlighted during the
arterial phase with washout during the venous phase, this reflects the hypervascularity of the lesions.The risk of tumour seeding as a result of a liver
biopsy is 2.7% with a median time interval between biopsy and seeding of 17 months[5]

Barcelona Clinic Liver Classification


There are many classification systems for addressing the management and prognosis, the BCLC system has the convenience of categorising disease
extent with treatment and prognostic outcomes. In determining the ideal treatment modality for HCC the key points are not just disease extent, but
also the functional state of the liver and patient.

Prognosis-5 yr
Stage Features Treatment survival

Stage Child-Pugh A Resection 40-70%


0 Single lesion (less than 2cm)
Normal portal pressures

Stage Single nodule greater than 3cm or multiple nodules (no If associated disease then radiofrequency ablation May be up to 70% in
A more than 3) If no associated disease then transplantation some
Child Pugh A/ B

Stage Multiple nodules Trans arterial chemo-embolisation (usually with 26% at 3 years
B Child Pugh A/B doxorubicin)

Stage Advanced tumours Sorafenib Usually survive 10.7


C Invasion of portal vein months
Child Pugh A/B

http://www.emrcs.com/question/question.php?q=0 1/3
eMRCS 19/07/2015

Stage Child Pugh stage C Best supportive care Less than 6 months
D Advanced tumours survival

[6]
In selected patients the best outcomes are achieved with surgical resection, or transplantation where surgical resection is precluded. Anatomical
resections with minimum 2cm margins provide the best outcomes.
At the present time there is no evidence to recommend treatment with adjuvant chemotherapy[6].

Sorafenib
This is an oral multi tyrosine kinase inhibitor. It is the only drug that has been currently demonstrated to extend survival in individuals with advanced
hepatocellular cancer[7]. The improvement in survival is from a median of 7 months to 10 months.

References
1. Jemal, A., et al., Global cancer statistics. CA Cancer J Clin, 2011. 61(2): p. 69-90.
2. Leese, T., O. Farges, and H. Bismuth, Liver cell adenomas. A 12-year surgical experience from a specialist hepato-biliary unit. Ann Surg, 1988.
208(5): p. 558-64.
3. Farges, O. and S. Dokmak, Malignant transformation of liver adenoma: an analysis of the literature. Dig Surg, 2010. 27(1): p. 32-8.
4. Ehrl, D., et al., "Incidentaloma" of the liver: management of a diagnostic and therapeutic dilemma. HPB Surg, 2012. 2012: p. 891787.
5. Silva, M.A., et al., Needle track seeding following biopsy of liver lesions in the diagnosis of hepatocellular cancer: a systematic review and meta-
analysis. Gut, 2008. 57(11): p. 1592-6.
6. EASL-EORTC clinical practice guidelines: management of hepatocellular carcinoma. J Hepatol, 2012. 56(4): p. 908-43.
7. Abou-Alfa, G.K., et al., Phase II study of sorafenib in patients with advanced hepatocellular carcinoma. J Clin Oncol, 2006. 24(26): p. 4293-300.

Save my notes

Please rate this question: Next question

Send feedback to the author

Question stats

A 25.6%
B 29.2%
C 24%
D 8.5%
E 12.8%

29.2% of users answered this question correctly

Search eMRCS

http://www.emrcs.com/question/question.php?q=0 2/3
eMRCS 19/07/2015

Search term Go

External links

Suggest a link

Dashboard

Overall score: 14.3%


1

3
4-6 0/3
7

All contents of this site are © 2015 E-Medical Revision Ltd Back to top

http://www.emrcs.com/question/question.php?q=0 3/3
eMRCS 19/07/2015

1/3
Question 8-10 of 22

Theme: Management of biliary diseases

A. Acute laparoscopic cholecystectomy


B. Delayed laparoscopic cholecystectomy
C. Percutaneous cholecystostomy
D. Lithotripsy
E. Endoscopic retrograde cholangiopancreatography
F. Choledochoduodenostomy
G. Bile duct excision and hepatico-jejunostomy
H. Operative cholecystostomy

Please select the most appropriate management option for the scenario given. Each option may be used once, more than once or not at all.

8. A 43 year old women is admitted with acute cholecystitis and fails to settle. A laparoscopic cholecystectomy is being performed, at
operation the gallbladder has evidence of an empyema and Calots triangle is inflamed and the surgeon suspects that a Mirizzi syndrome
has occurred.

You answered Lithotripsy

The correct answer is Operative cholecystostomy

This will address the acute sepsis and resolve the situation. Attempts at completing the surgery at this stage, even in expert hands carries
a very high risk of bile duct injury.

9. Following a difficult cholecystectomy a surgeon leaves a drain. 24 hours later bile is seen to be accumulating in the drain and this fails to
resolve over the next 48 hours. The patient is otherwise well.

You answered Operative cholecystostomy

The correct answer is Endoscopic retrograde cholangiopancreatography

This will delineate the presence of potential bile duct injury. Usually this is result of leakage from the cystic duct and placement of a stent
will allow free biliary drainage and the leak should settle.

10. A 40 year old woman is admitted with abdominal pain. She has suffered from repeated episodes of this colicky right upper quadrant pain.
On examination she is pyrexial with right upper quadrant peritonism. Her blood tests show a white cell count of 23. However, the liver
function tests are normal. An abdominal ultrasound scan shows multiple gallstones in a thick walled gallbladder, the bile duct measures
4mm.

Acute laparoscopic cholecystectomy

This lady has acute cholecystitis and needs an acute cholecystectomy. This operation should usually be performed within 48 hours of
admission. Delay beyond this timeframe will usually result in increased operative complications and most surgeons would administer
antibiotics and perform and interval cholecystectomy if the early window for an acute procedure is missed. A bile duct measuring 4mm is
usually normal.

Next question

Biliary disease

Diagnosis Typical features Pathogenesis

http://www.emrcs.com/question/question.php?q=0 1/3
eMRCS 19/07/2015

Gallstones Typically history of biliary colic Usually small calibre gallstones which can pass through the cystic duct. In Mirizzi
or episodes of chlolecystitis. syndrome the stone may compress the bile duct directly- one of the rare times that
Obstructive type history and cholecystitis may present with jaundice
test results.

Cholangitis Usually obstructive and will Ascending infection of the bile ducts usually by E. coli and by definition occurring in a
have Charcot's triad of pool of stagnant bile.
symptoms (pain, fever,
jaundice)

Pancreatic cancer Typically painless jaundice with Direct occlusion of distal bile duct or pancreatic duct by tumour. Sometimes nodal
palpable gallbladder disease at the portal hepatis may be the culprit in which case the bile duct may be of
(Courvoisier's Law) normal calibre.

TPN (total Usually follows long term use Often due to hepatic dysfunction and fatty liver which may occur with long term TPN
parenteral nutrition) and is usually painless with non usage.
associated jaundice obstructive features

Bile duct injury Depending upon the type of Often due to a difficult laparoscopic​ cholecystectomy when anatomy in Calots triangle is
injury may be of sudden or not appreciated. In the worst scenario the bile duct is excised and jaundice develops
gradual onset and is usually of rapidly post operatively. More insidious is that of bile duct stenosis which may be caused
obstructive type by clips or diathermy injury.

Cholangiocarcinoma Gradual onset obstructive Direct occlusion by disease and also extrinsic compression by nodal disease at the porta
pattern hepatis.

Septic surgical Usually hepatic features Combination of impaired biliary excretion and drugs such as ciprofloxacin which may
patient cause cholestasis.

Metastatic disease Mixed hepatic and post Combination of liver synthetic failure (late) and extrinsic compression by nodal disease
hepatic and anatomical compression of intra hepatic structures (earlier)

A gallbladder may develop a thickened wall in chronic cholecystitis, microscopically Roikitansky-Aschoff Sinuses may be seen

Image sourced from Wikipedia

Save my notes

Please rate this question: Next question

http://www.emrcs.com/question/question.php?q=0 2/3
eMRCS 19/07/2015

Send feedback to the author

Question stats

Average score for registered users:

8 41.7%
9 65.7%
10 59.6%

Search eMRCS

Search term Go

External links

Suggest a link

Dashboard

Overall score: 20%


1

3
4-6 0/3
7
8-10 1 / 3

All contents of this site are © 2015 E-Medical Revision Ltd Back to top

http://www.emrcs.com/question/question.php?q=0 3/3
eMRCS 19/07/2015

0/3
Question 11-13 of 22

Theme: Liver tumours

A. Rhabdomyosarcoma
B. Yolk sac tumour
C. Hepatocellular carcinoma
D. Metastatic lesion
E. Haemangioendothelioma
F. Cholangiocarcinoma
G. Hepatoblastoma
H. Angiosarcoma

Please select the most likely diagnosis for the scenario given. Each answer may be used once, more than once or not at all.

11. A 56 year old man with long standing ulcerative colitis and a DALM lesion in the rectum is admitted with jaundice. On CT scanning the liver
has 3 nodules in the right lobe and 1 nodule in the left lobe. Carcinoembryonic antigen levels are elevated.

You answered Hepatocellular carcinoma

The correct answer is Metastatic lesion

This is likely to be due to metastatic lesions from a colonic primary. DALM lesions should be excised by oncological colectomy for this
reason. This burden of metastatic disease is unlikely to precipitate jaundice directly and nodal disease at the porta hepatis is the most likely
cause in this case.

12. A 48 year old lady with chronic hepatitis B infection is noted to have worsening liver function tests and progressive jaundice. Her alpha feto
protein levels are grossly elevated.

You answered Hepatoblastoma

The correct answer is Hepatocellular carcinoma

This is most likely to be hepatocellular carcinoma and markedly elevated AFP levels in association with a compatible risk factor should make
this the diagnosis.

13. A 55 year old man with long standing ulcerative colitis is admitted with cholangitis and weight loss. Blood tests reveal a markedly elevated
CA 19-9.

You answered Metastatic lesion

The correct answer is Cholangiocarcinoma

This is most likely a cholangiocarcinoma. UC with sclerosing cholangitis increases the risk of cholangiocarcinoma. CA19-9 is elevated in
approximately 80% cases.

Next question

Liver tumours

Primary liver tumours


The most common primary tumours are cholangiocarcinoma and hepatocellular carcinoma. Overall metastatic disease accounts for 95% of all liver
malignancies making the primary liver tumours comparatively rare.

Primary liver tumours include:

http://www.emrcs.com/question/question.php?q=0 1/3
eMRCS 19/07/2015

Cholangiocarcinoma
Hepatocellular carcinoma
Hepatoblastoma
Sarcomas (Rare)
Lymphomas
Carcinoids (most often secondary although primary may occur)

Hepatocellular carcinoma
These account for the bulk of primary liver tumours (75% cases). Its worldwide incidence reflects its propensity to occur on a background of
chronic inflammatory activity. Most cases arise in cirrhotic livers or those with chronic hepatitis B infection, especially where viral replication is actively
occurring. In the UK it accounts for less than 5% of all cancers, although in parts of Asia its incidence is 100 per 100,000.
The majority of patients (80%) present with existing liver cirrhosis, with a mass discovered on screening ultrasound.

Diagnosis
CT/ MRI (usually both) are the imaging modalities of choice
a-fetoprotein is elevated in almost all cases
Biopsy should be avoided as it seeds tumours cells through a resection plane.
In cases of diagnostic doubt serial CT and αFP measurements are the preferred strategy.

Treatment
Patients should be staged with liver MRI and chest, abdomen and pelvic CT scan.
The testis should be examined in males (testicular tumours may cause raised AFP). PET CT may be used to identify occult nodal disease.
Surgical resection is the mainstay of treatment in operable cases. In patients with a small primary tumour in a cirrhotic liver whose primary
disease process is controlled, consideration may be given to primary whole liver resection and transplantation.
Liver resections are an option but since most cases occur in an already diseased liver the operative risks and post-operative hepatic
dysfunction are far greater than is seen following metastectomy.
These tumours are not particularly chemo or radiosensitive however, both may be used in a palliative setting. Tumour ablation is a more
popular strategy.

Survival
Poor, overall survival is 15% at 5 years.

Cholangiocarcinoma
This is the second most common type of primary liver malignancy. As its name suggests these tumours arise in the bile ducts. Up to 80% of
tumours arise in the extra hepatic biliary tree. Most patients present with jaundice and by this stage the majority will have disease that is not
resectable.
Primary sclerosing cholangitis is the main risk factor. In deprived countries typhoid and liver flukes are also major risk factors.

Diagnosis
Patients will typically have an obstructive picture on liver function tests.
CA 19-9, CEA and CA 125 are often elevated
CT/ MRI and MRCP are the imaging methods of choice.

Treatment
Surgical resection offers the best chance of cure. Local invasion of peri hilar tumours is a particular problem and this coupled with lobar
atrophy will often contra indicate surgical resection.
Palliation of jaundice is important, although metallic stents should be avoided in those considered for resection.

Survival
Is poor, approximately 5-10% 5 year survival.

Save my notes

http://www.emrcs.com/question/question.php?q=0 2/3
eMRCS 19/07/2015

Please rate this question: Next question

Send feedback to the author

Question stats

Average score for registered users:

11 74.8%
12 88.8%
13 82%

Search eMRCS

Search term Go

External links

Suggest a link

Dashboard

Overall score: 15.4%


1

3
4-6 0/3
7
8-10 1 / 3
11-
0/3
13

All contents of this site are © 2015 E-Medical Revision Ltd Back to top

http://www.emrcs.com/question/question.php?q=0 3/3
eMRCS 19/07/2015

Question 14 of 22

A 45 year old man presents with an episode of alcoholic pancreatitis. He makes slow but steady progress. He is reviewed clinically at 6 weeks following
admission. He has a diffuse fullness of his upper abdomen and on imaging a collection of fluid is found to be located behind the stomach. His serum
amylase is mildly elevated. Which of the following is the most likely explanation?

Early fluid collection

Pancreatic abscess

Peripancreatic necrosis

Pseudocyst

Sterile necrosis

Next question

Psuedocysts are unlikely to be present less than 4 weeks after an attack of acute pancreatitis. However, they are more common at this stage and
are associated with a raised amylase.

Pancreatitis: sequelae

Peripancreatic fluid collections


Occur in 25% cases
Located in or near the pancreas and lack a wall of granulation or fibrous tissue
May resolve or develop into pseudocysts or abscesses
Since most resolve aspiration and drainage is best avoided as it may precipitate infection

Pseudocysts
In acute pancreatitis result from organisation of peripancreatic fluid collection. They may or may not communicate with the ductal system.
The collection is walled by fibrous or granulation tissue and typically occurs 4 weeks or more after an attack of acute pancreatitis
Most are retrogastric
75% are associated with persistent mild elevation of amylase
Investigation is with CT, ERCP and MRI or Endoscopic USS
Symptomatic cases may be observed for 12 weeks as up to 50% resolve
Treatment is either with endoscopic or surgical cystogastrostomy or aspiration

Pancreatic necrosis
Pancreatic necrosis may involve both the pancreatic parenchyma and surrounding fat
Complications are directly linked to extent of parenchymal necrosis and extent of necrosis overall
Early necrosectomy is associated with a high mortality rate (and should be avoided unless compelling indications for surgery exist)
Sterile necrosis should be managed conservatively (at least initially)
Some centres will perform fine needle aspiration sampling of necrotic tissue if infection is suspected. False negatives may occur. The extent of
sepsis and organ dysfunction may be a better guide to surgery

Pancreatic abscess
Intra abdominal collection of pus associated with pancreas but in the absence of necrosis
Typically occur as a result of infected pseudocyst
They are usually managed by placement of percutaneous drains

Haemorrhage
Infected necrosis may involve vascular structures with resultant haemorrhage that may occur de novo or as a result of surgical
necrosectomy.
When retroperitoneal haemorrhage occurs Grey Turners sign may be identified

http://www.emrcs.com/question/question.php?q=0 1/3
eMRCS 19/07/2015

Save my notes

Please rate this question: Next question

Send feedback to the author

Question stats

A 8.2%
B 8.2%
C 10%
D 66%
E 7.6%

66% of users answered this question correctly

Search eMRCS

Search term Go

External links

Suggest a link

Dashboard

Overall score: 21.4%


1

http://www.emrcs.com/question/question.php?q=0 2/3
eMRCS 19/07/2015

3
4-6 0/3
7
8-10 1 / 3
11-13 0 / 3
14

All contents of this site are © 2015 E-Medical Revision Ltd Back to top

http://www.emrcs.com/question/question.php?q=0 3/3
eMRCS 19/07/2015

Question 15 of 22

A 34 year old lady is admitted with pancreatitis. The aetiology is unclear and it is classified as an attack of moderate severity according to the
Glasgow criteria. Her imaging shows no gallstones and fluid around the pancreas. Which of the following is the most appropriate initial management
option?

Laparotomy

Laparoscopy

Radiological aspiration of the fluid

Active observation

Administration of octreotide

Next question

LEARN THIS!

Mnemonic for the assessment of the severity of pancreatitis: PANCREAS


(Ann R Coll Surg Engl 2000; 82: 16-17

P a02 < 60 mmHg


A ge > 55 years
N eutrophils > 15 x 10/l
C alcium < 2 mmol/l
R aised urea > 16 mmol/l
E nzyme (lactate dehydrogenase) > 600 units/l
A lbumin < 32 g/l
S ugar (glucose) > 10 mmol/l

> 3 positive criteria indicates severe pancreatitis.

Acute early fluid collections are seen in 25% of patients with pancreatitis and require no specific treatment. Attempts at drainage may introduce
infection and result in pancreatic abscess formation.

Management of Pancreatitis

Management of Acute Pancreatitis in the UK

Diagnosis
Traditionally hyperamylasaemia has been utilised with amylase being elevated three times the normal range.
However, amylase may give both false positive and negative results.
Serum lipase is both more sensitive and specific than serum amylase. It also has a longer half life.
Serum amylase levels do not correlate with disease severity.

Differential causes of hyperamylasaemia


Acute pancreatitis

Pancreatic pseudocyst

Mesenteric infarct

Perforated viscus

http://www.emrcs.com/question/question.php?q=0 1/4
eMRCS 19/07/2015

Acute cholecystitis

Diabetic ketoacidosis

Assessment of severity
Glasgow, Ranson scoring systems and APACHE II
Biochemical scoring e.g. using CRP

Features that may predict a severe attack within 48 hours of admission to hospital
Initial assessment Clinical impression of severity
Body mass index >30
Pleural effusion
APACHE score >8

24 hours after admission Clinical impression of severity


APACHE II >8
Glasgow score of 3 or more
Persisting multiple organ failure
CRP>150

48 hours after admission Glasgow Score of >3


CRP >150
Persisting or progressive organ failure

Table adapted from UK guidelines for management of acute pancreatitis. GUT 2005, 54 suppl III

Management

Nutrition
There is reasonable evidence to suggest that the use of enteral nutrition does not worsen the outcome in pancreatitis
Most trials to date were underpowered to demonstrate a conclusive benefit.
The rationale behind feeding is that it helps to prevent bacterial translocation from the gut, thereby contributing to the development of
infected pancreatic necrosis.

Use of antibiotic therapy


Many UK surgeons administer antibiotics to patients with acute pancreatitis. However, there is very little evidence to support this practice.
A recent Cochrane review highlights the potential benefits of administering Imipenem to patients with established pancreatic necrosis in the
hope of averting the progression to infection.
There are concerns that the administration of antibiotics in mild attacks of pancreatitis will not affect outcome and may contribute to antibiotic
resistance and increase the risks of antibiotic associated diarrhoea.

Surgery
Patients with acute pancreatitis due to gallstones should undergo early cholecystectomy.
Patients with obstructed biliary system due to stones should undergo early ERCP.
Patients with extensive necrosis where infection is suspected should usually undergo FNA for culture.
Patients with infected necrosis should undergo either radiological drainage or surgical necrosectomy. The choice of procedure depends upon
local expertise.

References
www.bsg.org.uk/pdfworddocs/pancreatic.pdf

Antibiotic therapy for prophylaxis against infection of pancreatic necrosis in acute pancreatitis. Villatoro et al. Cochrane Library DOI:
10.1002/14651858.CD002941.pub3. 2010 version.

Save my notes

http://www.emrcs.com/question/question.php?q=0 2/4
eMRCS 19/07/2015

Please rate this question: Next question

Send feedback to the author

Question stats

A 6.6%
B 7.9%
C 12.8%
D 55.1%
E 17.7%

55.1% of users answered this question correctly

Search eMRCS

Search term Go

External links

Suggest a link

Dashboard

Overall score: 20%


1

3
4-6 0/3
7
8-10 1 / 3
11-13 0 / 3
14

15

http://www.emrcs.com/question/question.php?q=0 3/4
eMRCS 19/07/2015

All contents of this site are © 2015 E-Medical Revision Ltd Back to top

http://www.emrcs.com/question/question.php?q=0 4/4
eMRCS 19/07/2015

Question 16 of 22

A 43 year old lady presents with an attack of acute pancreatitis. It is classified as a mild attack on severity scoring. Imaging identifies gallstones but a
normal calibre bile duct, and a peripancreatic fluid collection. Which of the following management options is most appropriate?

Intravenous octreotide

Cholecystectomy within 4 weeks

Nasogastric tube drainage of the stomach

Insertion of a radiological drain

Avoidance of enteral feeding

Next question

Patients with gallstone pancreatitis should undergo early cholecystectomy.


Enteral feeding helps minimise gut bacterial translocation and should be given to most patients with pancreatitis. Many studies have evaluated the
role of octreotide in reducing pancreatic secretions and shown no benefit (Uhl W et al Gut 1999 45:97-104, McKay C et al. Int J Pancreatol 1997;
21: 13-19).
The use of antibiotics in pancreatitis is controversial. However, a recent Cochrane review has presented reasonable evidence in favor of
administration of imipenem to prevent infection in established necrosis.

Management of Pancreatitis

Management of Acute Pancreatitis in the UK

Diagnosis
Traditionally hyperamylasaemia has been utilised with amylase being elevated three times the normal range.
However, amylase may give both false positive and negative results.
Serum lipase is both more sensitive and specific than serum amylase. It also has a longer half life.
Serum amylase levels do not correlate with disease severity.

Differential causes of hyperamylasaemia


Acute pancreatitis

Pancreatic pseudocyst

Mesenteric infarct

Perforated viscus

Acute cholecystitis

Diabetic ketoacidosis

Assessment of severity
Glasgow, Ranson scoring systems and APACHE II
Biochemical scoring e.g. using CRP

Features that may predict a severe attack within 48 hours of admission to hospital
Initial assessment Clinical impression of severity
Body mass index >30
Pleural effusion

http://www.emrcs.com/question/question.php?q=0 1/3
eMRCS 19/07/2015

APACHE score >8

24 hours after admission Clinical impression of severity


APACHE II >8
Glasgow score of 3 or more
Persisting multiple organ failure
CRP>150

48 hours after admission Glasgow Score of >3


CRP >150
Persisting or progressive organ failure

Table adapted from UK guidelines for management of acute pancreatitis. GUT 2005, 54 suppl III

Management

Nutrition
There is reasonable evidence to suggest that the use of enteral nutrition does not worsen the outcome in pancreatitis
Most trials to date were underpowered to demonstrate a conclusive benefit.
The rationale behind feeding is that it helps to prevent bacterial translocation from the gut, thereby contributing to the development of
infected pancreatic necrosis.

Use of antibiotic therapy


Many UK surgeons administer antibiotics to patients with acute pancreatitis. However, there is very little evidence to support this practice.
A recent Cochrane review highlights the potential benefits of administering Imipenem to patients with established pancreatic necrosis in the
hope of averting the progression to infection.
There are concerns that the administration of antibiotics in mild attacks of pancreatitis will not affect outcome and may contribute to antibiotic
resistance and increase the risks of antibiotic associated diarrhoea.

Surgery
Patients with acute pancreatitis due to gallstones should undergo early cholecystectomy.
Patients with obstructed biliary system due to stones should undergo early ERCP.
Patients with extensive necrosis where infection is suspected should usually undergo FNA for culture.
Patients with infected necrosis should undergo either radiological drainage or surgical necrosectomy. The choice of procedure depends upon
local expertise.

References
www.bsg.org.uk/pdfworddocs/pancreatic.pdf

Antibiotic therapy for prophylaxis against infection of pancreatic necrosis in acute pancreatitis. Villatoro et al. Cochrane Library DOI:
10.1002/14651858.CD002941.pub3. 2010 version.

Save my notes

Please rate this question: Next question

http://www.emrcs.com/question/question.php?q=0 2/3
eMRCS 19/07/2015

Send feedback to the author

Question stats

A 12.9%
B 51%
C 11.3%
D 12.1%
E 12.7%

51% of users answered this question correctly

Search eMRCS

Search term Go

External links

Suggest a link

Dashboard

Overall score: 17.6%


1
2

3
4-6 0/3
7
8-10 1 / 3
11-13 0 / 3
14

15

16
17 -

All contents of this site are © 2015 E-Medical Revision Ltd Back to top

http://www.emrcs.com/question/question.php?q=0 3/3
eMRCS 19/07/2015

Question 17 of 22

A 43 year old lady with repeated episodes of abdominal pain is admitted with small bowel obstruction. A laparotomy is performed and at surgery a
gallstone ileus is identified. What is the most appropriate course of action?

Remove the gallstone via a proximally sited terminal ileal enterotomy and decompress the small bowel. Leave the gallbladder in situ.

Remove the gallstone via a proximally sited terminal ileal enterotomy and decompress the small bowel. Remove the gallbladder.

Remove the gallstone via a proximally sited terminal ileal enterotomy and decompress the small bowel. Perform a
choledochoduodenostomy.

Remove the gallstone from an enterotomy at the site of the obstruction and leave the gallbladder in situ.

Remove the gallstone from an enterotomy at the site of the obstruction and remove the gallbladder.

Next question

Gallstone ileus occurs as a result of the fistula developing between the gallbladder and the duodenum. These tend to become impacted somewhat
proximal to the ileocaecal valve and cause small bowel obstruction. The correct management is to remove the gallstone from an enterotomy
proximal to the site of stone impaction. The bowel at the site of impaction itself may not heal well and an enterotomy performed at this site may well
result in the need for a resection. The standard surgical teaching is that under almost all circumstances the gallbladder should be left in situ, as the
anatomy in this area is often hostile and unpredictable. Disconnecting it from the duodenum leaves a large defect that is difficult to close.

Gallstones

Up to 24% of women and 12% of men may have gallstones. Of these up to 30% may develop local infection and cholecystitis. In patients
subjected to surgery 12% will have stones contained within the common bile duct. The majority of gallstones are of a mixed composition (50%)
with pure cholesterol stones accounting for 20% of cases.
The aetiology of CBD stones differs in the world, in the West most CBD stones are the result of migration. In the East a far higher proportion arise in
the CBD de novo.
The classical symptoms are of colicky right upper quadrant pain that occurs post prandially. The symptoms are usually worst following a fatty meal
when cholecystokinin levels are highest and gallbladder contraction is maximal.

Investigation
In almost all suspected cases the standard diagnostic work up consists of abdominal ultrasound and liver function tests. Of patients who have stones
within the bile duct, 60% will have at least one abnormal result on LFT's. Ultrasound is an important test, but is operator dependent and therefore
may occasionally need to be repeated if a negative result is at odds with the clinical picture. Where stones are suspected in the bile duct, the options
lie between magnetic resonance cholangiography and intraoperative imaging. The choice between these two options is determined by the skills and
experience of the surgeon. The advantages of intra operative imaging are less useful in making therapeutic decisions if the operator is unhappy
about proceeding the bile duct exploration, and in such circumstances pre operative MRCP is probably a better option.

Specific gallstone and gallbladder related disease


Disease Features Management

Biliary colic Colicky abdominal pain, worse If imaging shows gallstones and history compatible then laparoscopic cholecystectomy
post prandially, worse after
fatty foods

Acute Right upper quadrant Imaging (USS) and cholecystectomy (ideally within 48 hours of presentation) (2)
cholecystitis pain
Fever
Murphys sign on
examination
Occasionally mildly
deranged LFT's
(especially if Mirizzi

http://www.emrcs.com/question/question.php?q=0 1/4
eMRCS 19/07/2015

syndrome)

Gallbladder Usually prodromal illness Imaging with USS +/- CT Scanning


abscess and right upper Ideally surgery, sub total cholecystectomy may be needed if Calots triangle is hostile
quadrant pain In unfit patients percutaneous drainage may be considered
Swinging pyrexia
Patient may be
systemically unwell
Generalised peritonism
not present

Cholangitis Patient severely septic Fluid resuscitation


and unwell Broad spectrum intravenous antibiotics
Jaundice Correct any coagulopathy
Right upper quadrant Early ERCP
pain

Gallstone Patients may have a Laparotomy and removal of gallstone from small bowel, the enterotomy must be made proximal
ileus history of previous to the site of obstruction and not at the site of obstruction. The fistula between the gallbladder
cholecystitis and known and duodenum should not be interfered with.
gallstones
Small bowel obstruction
(may be intermittent)

Acalculous Patients with inter If patient fit then cholecystectomy, if unfit then percutaneous cholecystostomy
cholecystitis current illness (e.g.
diabetes, organ failure)
Patient of systemically
unwell
Gallbladder inflammation
in absence of stones
High fever

Treatment
Patients with asymptomatic gallstones rarely develop symptoms related to them (less than 2% per year) and may therefore be managed
expectantly. In almost all cases of symptomatic gallstones the treatment of choice is cholecystectomy performed via the laparoscopic route. In the
very frail patient there is sometimes a role for selective use of ultrasound guided cholecystostomy.
During the course of the procedure some surgeons will routinely perform either intra operative cholangiography to either confirm anatomy or to
exclude CBD stones. The latter may be more easily achieved by use of laparoscopic ultrasound. If stones are found then the options lie between
early ERCP in the day or so following surgery or immediate surgical exploration of the bile duct. When performed via the trans cystic route this adds
little in the way of morbidity and certainly results in faster recovery. Where transcystic exploration fails the alternative strategy is that of formal
choledochotomy. The exploration of a small duct is challenging and ducts of less than 8mm should not be explored. Small stones that measure less
than 5mm may be safely left and most will pass spontaneously.

Risks of ERCP(1)
Bleeding 0.9% (rises to 1.5% if sphincterotomy performed)
Duodenal perforation 0.4%
Cholangitis 1.1%
Pancreatitis 1.5%

References
1. Williams E et al. Guidelines on the management of common bile duct stones (CBDS)Gut 2008;57:10041021

2. Gurusamy KS, Samraj K. Early versus delayed laparoscopic cholecystectomy for acute cholecystitis. Cochrane Database Syst Rev. 2006 Oct
18;(4):CD005440.

3. Gurusamy K and Davidson B. Gallstones. BMJ 2014 (348):27-30.

Save my notes

http://www.emrcs.com/question/question.php?q=0 2/4
eMRCS 19/07/2015

Please rate this question: Next question

Send feedback to the author

Question stats

A 28.2%
B 28.9%
C 15.2%
D 11.6%
E 16.2%

28.2% of users answered this question correctly

Search eMRCS

Search term Go

External links

Suggest a link

Dashboard

Overall score: 22.2%


1

2
3
4-6 0/3
7
8-10 1 / 3
11-13 0 / 3
14
15

http://www.emrcs.com/question/question.php?q=0 3/4
eMRCS 19/07/2015

16

17

All contents of this site are © 2015 E-Medical Revision Ltd Back to top

http://www.emrcs.com/question/question.php?q=0 4/4
eMRCS 19/07/2015

Question 18 of 22

A 43 year old lady presents with jaundice and is diagnosed as having a carcinoma of the head of the pancreas. Although she is deeply jaundiced, her
staging investigations are negative for metastatic disease. What is the best method of biliary decompression in this case?

ERCP and placement of metallic stent

ERCP alone

ERCP and placement of plastic stent

Cholecystostomy

Choledochoduodenostomy

Next question

Metallic stents are contraindicated in resectable biliary disease

A plastic stent is the best option for biliary decompression in resectable disease. Surgical bypasses have no place in the management of operable
malignancy as a bridge to definitive surgery.

Pancreatic stents

Both benign and malignant biliary obstruction may be treated by placement of stents. These may be either plastic tubes or self expanding metallic
stents. They can be placed either percutaneously, at ERCP, or, less commonly now, open surgery. Complications include blockage, displacement
and those related to the method of insertion.

Metallic Vs Plastic stents


Metallic stents Plastic stents

Expensive Cheap

Embed in surrounding tissues Do not usually embed

Displacement rare Displacement common

Blockage rare Blockage common

Contraindicated in resectable malignant disease May be used as a bridge to resectional surgery

Save my notes

http://www.emrcs.com/question/question.php?q=0 1/3
eMRCS 19/07/2015

Please rate this question: Next question

Send feedback to the author

Question stats

A 27.6%
B 7.2%
C 41.4%
D 6.1%
E 17.7%

41.4% of users answered this question correctly

Search eMRCS

Search term Go

External links

Suggest a link

Dashboard

Overall score: 21.1%


1

2
3
4-6 0/3
7
8-10 1 / 3
11-13 0 / 3
14
15

16

17

http://www.emrcs.com/question/question.php?q=0 2/3
eMRCS 19/07/2015

18

All contents of this site are © 2015 E-Medical Revision Ltd Back to top

http://www.emrcs.com/question/question.php?q=0 3/3
eMRCS 19/07/2015

0/3
Question 19-21 of 22

Theme: Pancreatitis management

A. Pancreatic necrosectomy
B. Staging laparotomy to assess severity
C. Endoscopic retrograde cholangiopancreatography
D. Emergency cystogastrostomy
E. Cholecystectomy within 4 weeks
F. Elective cystogastrostomy
G. Parenteral nutrition

Please select the most appropriate next stage in management for the scenario given. Each option may be used once, more than once or not at all.

19. A 34 year old women is admitted with cholangitis. Her bilirubin is 180 and alkaline phosphatase is 348. She becomes progressively more
unwell and develops abdominal pain. The houseman checks her amylase which is elevated at 1080. Standard treatment is initiated and her
Glasgow score is 3.

You answered Emergency cystogastrostomy

The correct answer is Endoscopic retrograde cholangiopancreatography

She requires urgent decompression of her biliary system. An ERCP is the conventional method of performing this. It is important to ensure
that her coagulation status is normalised prior to performing this procedure.

20. A 63 year old man is admitted to ITU with an attack of severe gallstone pancreatitis. He requires ventillatory support for ARDS. Over the
past few days he has become more unwell and a CT scan is organised. This demonstrates an area of necrosis, culture from this area shows
a gram negative bacillus. His CRP is 400 and WCC 25.1.

You answered Parenteral nutrition

The correct answer is Pancreatic necrosectomy

This man requires necrosectomy as he has infected pancreatic necrosis and is haemodynamically unstable. A radiological drainage
procedure is unlikely to be sufficient.

21. A 53 year old alcoholic develops acute pancreatitis and is making slow but reasonable progress. He is troubled by persisting ileus and for
this reason a CT scan is undertaken. This demonstrates a large pancreatic pseudocyst. This is monitored by repeat CT scanning which
shows no resolution and he is now complaining of early satiety.

You answered Pancreatic necrosectomy

The correct answer is Elective cystogastrostomy

Drainage of this man's pseudocyst is required. This could be accomplished radiologically or endoscopically or surgically. As the other
options are not on the list this is the best option from those available.

Next question

Management of Pancreatitis

Management of Acute Pancreatitis in the UK

Diagnosis
Traditionally hyperamylasaemia has been utilised with amylase being elevated three times the normal range.

http://www.emrcs.com/question/question.php?q=0 1/4
eMRCS 19/07/2015

However, amylase may give both false positive and negative results.
Serum lipase is both more sensitive and specific than serum amylase. It also has a longer half life.
Serum amylase levels do not correlate with disease severity.

Differential causes of hyperamylasaemia


Acute pancreatitis

Pancreatic pseudocyst

Mesenteric infarct

Perforated viscus

Acute cholecystitis

Diabetic ketoacidosis

Assessment of severity
Glasgow, Ranson scoring systems and APACHE II
Biochemical scoring e.g. using CRP

Features that may predict a severe attack within 48 hours of admission to hospital
Initial assessment Clinical impression of severity
Body mass index >30
Pleural effusion
APACHE score >8

24 hours after admission Clinical impression of severity


APACHE II >8
Glasgow score of 3 or more
Persisting multiple organ failure
CRP>150

48 hours after admission Glasgow Score of >3


CRP >150
Persisting or progressive organ failure

Table adapted from UK guidelines for management of acute pancreatitis. GUT 2005, 54 suppl III

Management

Nutrition
There is reasonable evidence to suggest that the use of enteral nutrition does not worsen the outcome in pancreatitis
Most trials to date were underpowered to demonstrate a conclusive benefit.
The rationale behind feeding is that it helps to prevent bacterial translocation from the gut, thereby contributing to the development of
infected pancreatic necrosis.

Use of antibiotic therapy


Many UK surgeons administer antibiotics to patients with acute pancreatitis. However, there is very little evidence to support this practice.
A recent Cochrane review highlights the potential benefits of administering Imipenem to patients with established pancreatic necrosis in the
hope of averting the progression to infection.
There are concerns that the administration of antibiotics in mild attacks of pancreatitis will not affect outcome and may contribute to antibiotic
resistance and increase the risks of antibiotic associated diarrhoea.

Surgery
Patients with acute pancreatitis due to gallstones should undergo early cholecystectomy.
Patients with obstructed biliary system due to stones should undergo early ERCP.
Patients with extensive necrosis where infection is suspected should usually undergo FNA for culture.
Patients with infected necrosis should undergo either radiological drainage or surgical necrosectomy. The choice of procedure depends upon
local expertise.

References
www.bsg.org.uk/pdfworddocs/pancreatic.pdf

Antibiotic therapy for prophylaxis against infection of pancreatic necrosis in acute pancreatitis. Villatoro et al. Cochrane Library DOI:

http://www.emrcs.com/question/question.php?q=0 2/4
eMRCS 19/07/2015

10.1002/14651858.CD002941.pub3. 2010 version.

Save my notes

Please rate this question: Next question

Send feedback to the author

Question stats

Average score for registered users:

19 68.5%
20 83.8%
21 50.8%

Search eMRCS

Search term Go

External links

Suggest a link

Dashboard

Overall score: 18.2%


1

3
4-6 0/3

http://www.emrcs.com/question/question.php?q=0 3/4
eMRCS 19/07/2015

7
8-10 1 / 3
11-13 0 / 3
14

15

16

17

18
19-
0/3
21

All contents of this site are © 2015 E-Medical Revision Ltd Back to top

http://www.emrcs.com/question/question.php?q=0 4/4
eMRCS 19/07/2015

Question 22 of 22

What proportion of patients presenting for cholecystectomy for treatment of biliary colic due to gallstones will have stones in the common bile duct?

10%

30%

2%

50%

25%

Up to 10% of all patients may have stones in the CBD. Therefore, all patients should have their liver function tests checked prior to embarking on a
cholecystectomy.

Biliary disease

Diagnosis Typical features Pathogenesis

Gallstones Typically history of biliary colic Usually small calibre gallstones which can pass through the cystic duct. In Mirizzi
or episodes of chlolecystitis. syndrome the stone may compress the bile duct directly- one of the rare times that
Obstructive type history and cholecystitis may present with jaundice
test results.

Cholangitis Usually obstructive and will Ascending infection of the bile ducts usually by E. coli and by definition occurring in a
have Charcot's triad of pool of stagnant bile.
symptoms (pain, fever,
jaundice)

Pancreatic cancer Typically painless jaundice with Direct occlusion of distal bile duct or pancreatic duct by tumour. Sometimes nodal
palpable gallbladder disease at the portal hepatis may be the culprit in which case the bile duct may be of
(Courvoisier's Law) normal calibre.

TPN (total Usually follows long term use Often due to hepatic dysfunction and fatty liver which may occur with long term TPN
parenteral nutrition) and is usually painless with non usage.
associated jaundice obstructive features

Bile duct injury Depending upon the type of Often due to a difficult laparoscopic​ cholecystectomy when anatomy in Calots triangle is
injury may be of sudden or not appreciated. In the worst scenario the bile duct is excised and jaundice develops
gradual onset and is usually of rapidly post operatively. More insidious is that of bile duct stenosis which may be caused
obstructive type by clips or diathermy injury.

Cholangiocarcinoma Gradual onset obstructive Direct occlusion by disease and also extrinsic compression by nodal disease at the porta
pattern hepatis.

Septic surgical Usually hepatic features Combination of impaired biliary excretion and drugs such as ciprofloxacin which may
patient cause cholestasis.

Metastatic disease Mixed hepatic and post Combination of liver synthetic failure (late) and extrinsic compression by nodal disease
hepatic and anatomical compression of intra hepatic structures (earlier)

A gallbladder may develop a thickened wall in chronic cholecystitis, microscopically Roikitansky-Aschoff Sinuses may be seen

http://www.emrcs.com/question/question.php?q=0 1/3
eMRCS 19/07/2015

Image sourced from Wikipedia

Save my notes

Please rate this question:

Send feedback to the author

Question stats

A 42.3%
B 15%
C 18.2%
D 9.3%
E 15.3%

42.3% of users answered this question correctly

Search eMRCS

Search term Go

http://www.emrcs.com/question/question.php?q=0 2/3
eMRCS 19/07/2015

External links

Suggest a link

Dashboard

Overall score: 17.4%


1

2
3
4-6 0/3
7
8-10 1 / 3
11-13 0 / 3
14
15

16

17
18
19-21 0 / 3
22

All contents of this site are © 2015 E-Medical Revision Ltd Back to top

http://www.emrcs.com/question/question.php?q=0 3/3

You might also like